LSAT and Law School Admissions Forum

Get expert LSAT preparation and law school admissions advice from PowerScore Test Preparation.

 180nce
  • Posts: 12
  • Joined: May 18, 2015
|
#19021
I was down to two answer choices for this question, but I ended up choosing (E), the wrong choice

I see why (A) weakens the argument, but why doesn't (E) weaken the argument? Isn't (E) saying that previously reviewers denied patents to new animal varieties created through conventional breeding programs and NOT denying patents created through genetic engineering.

Help please!

Thanks in advance
 Steve Stein
PowerScore Staff
  • PowerScore Staff
  • Posts: 1153
  • Joined: Apr 11, 2011
|
#19022
Hi 180,

Interesting question. The patent reviewers deny the patent, based on the prohibition regarding new animal varieties. The question asks for the answer choice that most weakens this argument, and as you mentioned, answer choice (A) does so; if those prohibitions pertain ONLY to domesticated farm animals, then they wouldn't apply in this case.

As for answer choice (E), it provides that they previously denied patents in other, different cases. You may have been drawn to this answer by the fact that it doesn't strengthen the reviewers' argument, but it doesn't weaken it either. This is a sometimes subtle but important distinction: a failure to strengthen does not necessarily weaken.

I hope that's helpful! Please let me know whether this is clear--thanks!

~Steve
 LustingFor!L
  • Posts: 80
  • Joined: Aug 27, 2016
|
#38065
I ruled out A and chose E. Looking back, E attacks the first sentence (premise), while A attacks the conclusion (second sentence). So I should have seen that and picked A, since most weaken questions leave premise untouched and attack conclusion/assumptions by author?
 AthenaDalton
PowerScore Staff
  • PowerScore Staff
  • Posts: 296
  • Joined: May 02, 2017
|
#38384
Hi Lathlee,

It is possible to weaken an argument by attacking one of its underlying premises. I don't think there is a hard-and-fast formula you can use to identify which answer choice weakens the argument beyond considering each one individually. As you know, only one answer choice will be correct. If, however, there were two correct answer choices -- one that weakens the conclusion, and one that weakens the premise -- the one that weakens the conclusion would be the stronger attack. However, since this is the world of the LSAT, there will only be one correct answer that weakens the argument, and it is possible that the answer choice weakens the argument by attacking a premise.

Here, answer choice (E) really doesn't weaken the argument at all. If the patent office has previously declined to grant patents to new animal varieties, this fact does nothing to indicate that they should now change their position and start granting patents for new animal varieties. If anything, evidence that they have ruled against patenting new animal varieties in the past strengthens the argument that they should continue to deny patents for new animal varieties. This isn't a great argument since, as Steve points out, the two animal varieties at issue are quite different.

I hope this makes sense. Good luck studying!

Get the most out of your LSAT Prep Plus subscription.

Analyze and track your performance with our Testing and Analytics Package.